K
Khách

Hãy nhập câu hỏi của bạn vào đây, nếu là tài khoản VIP, bạn sẽ được ưu tiên trả lời.

11 tháng 6 2017

à thôi ko cần làm bài này nữa

11 tháng 6 2017

TUI XOA NHE

29 tháng 1 2022

Có \(P=\dfrac{x+z}{xyz}=\dfrac{1}{yz}+\dfrac{1}{xy}=\dfrac{1}{y}\left(\dfrac{1}{x}+\dfrac{1}{z}\right)\ge\dfrac{1}{y}.\dfrac{4}{x+z}\)

\(=\dfrac{4}{y\left(x+z\right)}=\dfrac{4}{y\left(4-y\right)}=\dfrac{4}{-y^2+4y}=\dfrac{4}{-\left(y-2\right)^2+4}\ge1\)

"=" xảy ra khi y = 2 ; x = 1 ; z = 1

29 tháng 1 2022

Giúp mình câu này với ah.

 

4 tháng 10 2017

Bạn tham khảo nhé:

Ta có \(xyz=1\Rightarrow x+y+z\ge3\)

Áp dụng BĐT sờ- swat,ta có:

\(Q\ge\frac{9}{2\left(x+y+z\right)+3}\le1\)(vì \(x+y+z\ge3\))

Vậy max=1

4 tháng 10 2017

Hình như bài này mình bị nghịch dấu rồi

7 tháng 9 2021

\(4=x+y+z\ge3\sqrt[3]{xyz}\Leftrightarrow\sqrt[3]{xyz}\le\dfrac{4}{3}\Leftrightarrow xyz\le\dfrac{64}{27}\)(BĐT cauchy)

Dấu \("="\Leftrightarrow x=y=z=\dfrac{4}{3}\)

AH
Akai Haruma
Giáo viên
7 tháng 9 2021

Lời giải:

Áp dụng BĐT AM-GM:
$xy\le \frac{(x+y)^2}{4}=\frac{(4-z)^2}{4}$

$\Rightarrow H\leq \frac{z(4-z)^2}{4}$

Tiếp tục áp dụng BĐT AM-GM:
$z(4-z)\leq \frac{(z+4-z)^2}{4}=4$

$4-z\leq 2$ do $z\geq 2$

$\Rightarrow \frac{z(4-z)^2}{4}\leq \frac{4.2}{4}=2$

Hay $H\leq 2$ 

Vậy $H_{\max}=2$ khi $(x,y,z)=(1,1,2)$

13 tháng 5 2017

\(xy+yz+zx-xyz=1-x-y-z+xy+yz+zx-xyz\)

\(=\left(1-x\right)-y\left(1-x\right)-z\left(1-x\right)+yz\left(1-x\right)\)

\(=\left(1-x\right)\left(1-y-z+yz\right)=\left(1-x\right)\left(1-y\right)\left(1-z\right)\)

\(xy+yz+zx+xyz+2=1+x+y+z+xy+yz+zx+xyz\)

\(=\left(1+x\right)+y\left(1+x\right)+z\left(1+x\right)+yz\left(1+x\right)\)

\(=\left(1+x\right)\left(1+y\right)\left(1+z\right)\)

\(1+x+y+z=1+1\Rightarrow1+x=\left(1-y\right)+\left(1-z\right)\ge2\sqrt{\left(1-y\right)\left(1-z\right)}\)

Tương tự ta cũng có: \(1+y\ge2\sqrt{\left(1-z\right)\left(1-x\right)}\)

\(1+z\ge2\sqrt{\left(1-x\right)\left(1-y\right)}\)

Vậy \(S\le\frac{\left(1-x\right)\left(1-y\right)\left(1-z\right)}{8\left(1-x\right)\left(1-y\right)\left(1-z\right)}=\frac{1}{8}\)

28 tháng 9 2021

Tham khảo:

Cho 3 số thức x,y,z thỏa mãn \(x\ge1;y\ge4;z\ge9\) tìm giá trị lớn nhất của biết thức Q=\(\dfrac{yz\sqrt{x-1}+zx\sqrt... - Hoc24